MSRA Exam Practice Questions and Answers
Latest Updated| 100% Correct
QUESTION 1
A 45-year-old man presents with crushing central chest pain radiating to his left arm for 30
minutes. ECG shows ST-segment elevation in leads II, III, and aVF.
Which coronary artery is most likely occluded?
A. Left anterior descending artery
B. Left circumflex artery
C. Right coronary artery
D. Posterior descending artery
CORRECT ANSWER: C
RATIONALE: ST elevation in inferior leads (II, III, aVF) indicates an inferior myocardial infarction,
typically due to occlusion of the right coronary artery. The left anterior descending affects
anterior leads (V1–V4), and the circumflex affects lateral leads (I, aVL, V5–V6).
QUESTION 2
A 32-year-old woman presents with malaise, sore throat, and post-auricular lymphadenopathy.
She reports having a faint maculopapular rash.
Which pathogen is most likely responsible for her illness?
A. Epstein–Barr virus
B. Rubella virus
C. Cytomegalovirus
D. Parvovirus B19
CORRECT ANSWER: B
RATIONALE: Rubella presents with mild fever, lymphadenopathy (particularly post-auricular and
suboccipital), and a fine maculopapular rash spreading from face to trunk. EBV causes severe
fatigue with generalized lymphadenopathy, not typically post-auricular.
,QUESTION 3
A 70-year-old man complains of difficulty initiating urination and a weak stream. Digital rectal
examination reveals a smooth, enlarged prostate.
What is the most appropriate initial management?
A. Transurethral resection of the prostate
B. Finasteride
C. Alpha-1 adrenergic receptor blocker
D. Immediate catheterization
CORRECT ANSWER: C
RATIONALE: Alpha-1 blockers (e.g., tamsulosin) relieve dynamic obstruction from smooth muscle
tone in benign prostatic hyperplasia. Finasteride (a 5-alpha-reductase inhibitor) takes months to
work and TURP is reserved for resistant cases.
QUESTION 4
A 26-year-old woman presents with weight loss and heat intolerance. TSH is suppressed, and
free T4 is elevated.
Which antibody is most specific for her condition?
A. Anti-thyroid peroxidase antibody
B. Anti-thyroglobulin antibody
C. TSH receptor stimulating antibody
D. Anti-microsomal antibody
CORRECT ANSWER: C
RATIONALE: TSH-receptor stimulating antibodies are specific for Graves’ disease, causing diffuse
goiter and hyperthyroidism. Anti-thyroid peroxidase and anti-thyroglobulin are typical of
Hashimoto’s thyroiditis.
QUESTION 5
A 60-year-old smoker complains of chronic productive cough for at least three months in two
consecutive years.
, What is the most likely diagnosis?
A. Chronic bronchitis
B. Emphysema
C. Bronchiectasis
D. COPD with predominant emphysema
CORRECT ANSWER: A
RATIONALE: Chronic bronchitis is defined clinically as a productive cough for at least three
months in two consecutive years. Emphysema presents with dyspnea and minimal sputum.
QUESTION 6
A 35-year-old man presents with jaundice and right upper quadrant tenderness following
intravenous drug use. ALT and AST are markedly raised.
Which investigation confirms the diagnosis?
A. HBsAg serology
B. Anti-HBc IgG
C. Anti-HCV antibodies
D. Hepatitis A IgM
CORRECT ANSWER: C
RATIONALE: Anti-HCV antibody testing confirms current or past hepatitis C infection, which is
common in people who inject drugs. HBsAg indicates hepatitis B, and HAV IgM indicates acute
hepatitis A.
QUESTION 7
A 67-year-old man is admitted with a swollen, tender calf and recent long-haul flight.
What is the most appropriate initial investigation?
A. D-dimer test
B. Venous duplex ultrasound
C. CT pulmonary angiography
D. MRI venogram
CORRECT ANSWER: B
Latest Updated| 100% Correct
QUESTION 1
A 45-year-old man presents with crushing central chest pain radiating to his left arm for 30
minutes. ECG shows ST-segment elevation in leads II, III, and aVF.
Which coronary artery is most likely occluded?
A. Left anterior descending artery
B. Left circumflex artery
C. Right coronary artery
D. Posterior descending artery
CORRECT ANSWER: C
RATIONALE: ST elevation in inferior leads (II, III, aVF) indicates an inferior myocardial infarction,
typically due to occlusion of the right coronary artery. The left anterior descending affects
anterior leads (V1–V4), and the circumflex affects lateral leads (I, aVL, V5–V6).
QUESTION 2
A 32-year-old woman presents with malaise, sore throat, and post-auricular lymphadenopathy.
She reports having a faint maculopapular rash.
Which pathogen is most likely responsible for her illness?
A. Epstein–Barr virus
B. Rubella virus
C. Cytomegalovirus
D. Parvovirus B19
CORRECT ANSWER: B
RATIONALE: Rubella presents with mild fever, lymphadenopathy (particularly post-auricular and
suboccipital), and a fine maculopapular rash spreading from face to trunk. EBV causes severe
fatigue with generalized lymphadenopathy, not typically post-auricular.
,QUESTION 3
A 70-year-old man complains of difficulty initiating urination and a weak stream. Digital rectal
examination reveals a smooth, enlarged prostate.
What is the most appropriate initial management?
A. Transurethral resection of the prostate
B. Finasteride
C. Alpha-1 adrenergic receptor blocker
D. Immediate catheterization
CORRECT ANSWER: C
RATIONALE: Alpha-1 blockers (e.g., tamsulosin) relieve dynamic obstruction from smooth muscle
tone in benign prostatic hyperplasia. Finasteride (a 5-alpha-reductase inhibitor) takes months to
work and TURP is reserved for resistant cases.
QUESTION 4
A 26-year-old woman presents with weight loss and heat intolerance. TSH is suppressed, and
free T4 is elevated.
Which antibody is most specific for her condition?
A. Anti-thyroid peroxidase antibody
B. Anti-thyroglobulin antibody
C. TSH receptor stimulating antibody
D. Anti-microsomal antibody
CORRECT ANSWER: C
RATIONALE: TSH-receptor stimulating antibodies are specific for Graves’ disease, causing diffuse
goiter and hyperthyroidism. Anti-thyroid peroxidase and anti-thyroglobulin are typical of
Hashimoto’s thyroiditis.
QUESTION 5
A 60-year-old smoker complains of chronic productive cough for at least three months in two
consecutive years.
, What is the most likely diagnosis?
A. Chronic bronchitis
B. Emphysema
C. Bronchiectasis
D. COPD with predominant emphysema
CORRECT ANSWER: A
RATIONALE: Chronic bronchitis is defined clinically as a productive cough for at least three
months in two consecutive years. Emphysema presents with dyspnea and minimal sputum.
QUESTION 6
A 35-year-old man presents with jaundice and right upper quadrant tenderness following
intravenous drug use. ALT and AST are markedly raised.
Which investigation confirms the diagnosis?
A. HBsAg serology
B. Anti-HBc IgG
C. Anti-HCV antibodies
D. Hepatitis A IgM
CORRECT ANSWER: C
RATIONALE: Anti-HCV antibody testing confirms current or past hepatitis C infection, which is
common in people who inject drugs. HBsAg indicates hepatitis B, and HAV IgM indicates acute
hepatitis A.
QUESTION 7
A 67-year-old man is admitted with a swollen, tender calf and recent long-haul flight.
What is the most appropriate initial investigation?
A. D-dimer test
B. Venous duplex ultrasound
C. CT pulmonary angiography
D. MRI venogram
CORRECT ANSWER: B